Difference between revisions of "Schwarzschild black hole"

From Universe in Problems
Jump to: navigation, search
(Orbital motion, effective potential)
(Miscellaneous problems)
Line 827: Line 827:
 
<div id="BlackHole40"></div>
 
<div id="BlackHole40"></div>
 
=== Problem 27. ===
 
=== Problem 27. ===
problem formulation
+
Gravitational lensing is the effect of deflection of a light beam's (photon's) trajectory in the gravitational field. Derive the deflection of a photon's trajectory in Schwarzschild metric in the limit $L/r_{g}\gg 1$. Show that it is twice the value for a massive particle with velocity close to $c$ in the Newtonian theory.
 +
 
 
<div class="NavFrame collapsed">
 
<div class="NavFrame collapsed">
 
   <div class="NavHead">solution</div>
 
   <div class="NavHead">solution</div>
 
   <div style="width:100%;" class="NavContent">
 
   <div style="width:100%;" class="NavContent">
 
     <p style="text-align: left;">
 
     <p style="text-align: left;">
problem solution </p>
+
We choose $\lambda$ for massless particles as before and obtain
 +
\[\Big(\frac{dr}{d\lambda}\Big)^{2}+
 +
b^{2}\frac{h}{r^2}=1;\qquad
 +
\frac{d\varphi}{d\lambda}=b\frac{1}{r^2}.\]
 +
Excluding $\lambda$, after integration we obtain $\varphi(r)$. We are interested here in the variation of angle when $r$ goes from infinity to the minimal value, fro which the square root is zero. In the Newtonian theory it is $\pi/2$, so looking for the first correction to this value, we write
 +
\[\Delta\varphi\big|_{\pi/2}=
 +
b\int\limits_{r_{min}}^{\infty}
 +
\frac{dr/r^2} {\sqrt{1-hb^{2}/r^2}}=
 +
\left\|
 +
x=\frac{b}{r},\;\varepsilon=\frac{r_{g}}{r}\ll 1
 +
\right\|=
 +
\int\limits_{0}^{x_{max}}
 +
\frac{dx}{\sqrt{1-x^2+\varepsilon x^3}}.\]
 +
 
 +
Changing variables as $x^{2}-\varepsilon x^3=y^2$ and using the small parameter $\varepsilon$, we can transform the integral to one that is easy to compute
 +
\[\Delta\varphi\big|_{\pi/2}\approx\int\limits_{0}^{1}
 +
\frac{dy}{\sqrt{1-y^2}}(1+\varepsilon y)=
 +
\frac{\pi}{2}+\varepsilon=
 +
\frac{\pi}{2}+\frac{r_{g}}{b}.\]
 +
The second term is the needed correction, which is half the full correction $\delta\phi$ to the angle's variation when a particles moves from infinity and back to infinity:
 +
\[\delta\varphi\big|_{\pi}=\frac{2r_{g}}{b}=
 +
\frac{4GM}{b c^2}.\]
 +
 
 +
Now let us calculate the same thing in the Newtonian theory for a fast particle. Integrals of motion are
 +
\begin{align*}
 +
&L=b v_{\infty}=r^{2}\dot{\varphi},\\
 +
&E=U+\frac{\dot{r}^2}{2}+\frac{(r\dot{\varphi})^2}{2}
 +
=\frac{\dot{r}^2}{2}-\frac{r_g}{2r}+\frac{L^2}{2r^2}.
 +
\end{align*}
 +
Then $\varphi(r)$ can be written as
 +
\[\varphi=L\int\frac{dr/r^2}
 +
{\sqrt{2E+\frac{r_g}{r}-\frac{L^2}{r^2}}},\]
 +
and changing variables to $u=r^{-1}$, we obtain for the variation of angle from infinity to the turning point
 +
\[\varphi|_{\pi/2}=\int\limits_{0}^{u_{max}}\frac{du}
 +
{\sqrt{b^{-2}-u^2+\frac{r_g}{L^2}u}}.\]
 +
The last term under the root is a small correction. Let us make another change of variables $u=u'+\varepsilon/2$, where $\varepsilon=r_{g}/L^2 \ll 1$. Then up to terms of higher order by $\varepsilon$ the integral is reduced to
 +
\[\varphi|_{\pi/2}=\int\limits_{-\varepsilon/2}^{b^{-1}}
 +
\frac{du'}{\sqrt{b^{-2}-{u'}^2}}=
 +
\int\limits_{-b\varepsilon/2}^{1}
 +
\frac{d\xi}{\sqrt{1-\xi^2}}
 +
=\frac{\pi}{2}+\frac{b\varepsilon}{2}.\]
 +
Then for motion from infinity to infinity the deflection of the trajectory from a straight line is
 +
\[\delta\varphi|_{\pi}=b\varepsilon=
 +
\frac{b r_g}{L^2}
 +
=\frac{r_g}{b}\frac{1}{v_{\infty}^2}
 +
\to \frac{r_g}{b}.\]
 +
This is exactly half the correct quantity given by GTR.</p>
 
   </div>
 
   </div>
 
</div>
 
</div>
Line 838: Line 885:
 
<div id="BlackHole41"></div>
 
<div id="BlackHole41"></div>
 
=== Problem 28. ===
 
=== Problem 28. ===
problem formulation
+
Show that the $4$-acceleration of a stationary particle in the Schwarzschild metric can be presented in the form
 +
\[a_{\mu}=-\partial_{\mu}\Phi,\quad
 +
\text{where}\quad \Phi=\ln \sqrt{g_{00}}
 +
=\tfrac{1}{2}\ln g_{00}\]
 +
is some generalization of the Newtonian gravitational potential.
 
<div class="NavFrame collapsed">
 
<div class="NavFrame collapsed">
 
   <div class="NavHead">solution</div>
 
   <div class="NavHead">solution</div>
 
   <div style="width:100%;" class="NavContent">
 
   <div style="width:100%;" class="NavContent">
 
     <p style="text-align: left;">
 
     <p style="text-align: left;">
problem solution </p>
+
For a static particle $u^{\mu}=(u^0,0,0,0)$, where $u^0$ is found from the normalizing condition
 +
\[1=u^\mu u_\mu = g_{00}(u^{0})^2\quad
 +
\Rightarrow\quad u^{0}=\frac{1}{\sqrt{g_{00}}}.\]
 +
Then $4$-acceleration is
 +
\[a^\mu=\frac{du^\mu}{ds}
 +
=-\Gamma^{\mu}_{\nu\lambda}u^{\nu}u^{\lambda}
 +
=-\Gamma^{\mu}_{00}(u^0)^{2}\]
 +
and
 +
\[a_{\mu}=-\frac{1}{g_{00}}\Gamma_{\mu\,00}
 +
=-\frac{1}{2g_{00}}(-\partial_{\mu}g_{00})
 +
=\frac{1}{2}\frac{\partial_{\mu}g_{00}}{g_{00}}
 +
=\partial_{\mu}\Phi,\]
 +
where $\Phi=\ln\sqrt{g_{00}}$. </p>
 
   </div>
 
   </div>
 
</div>
 
</div>
Line 849: Line 912:
 
<div id="BlackHole42"></div>
 
<div id="BlackHole42"></div>
 
=== Problem 29. ===
 
=== Problem 29. ===
problem formulation
+
Let us reformulate the problem in a coordinate-independent manner. Suppose we have an arbitrary stationary metric with timelike Killing vector $\xi^\mu$, and we denote the $4$-velocity of a stationary observer by $u^{\mu}=\xi^{\mu}/V$. What is the $4$-force per unit mass that we need to apply to a test particle in order to make it stay stationary? Show in coordinate-independent way that the answer coincides with $\partial_{\mu}\Phi$ (up to the sign), and rewrite $\Phi$ in coordinate-independent form.
 
<div class="NavFrame collapsed">
 
<div class="NavFrame collapsed">
 
   <div class="NavHead">solution</div>
 
   <div class="NavHead">solution</div>
 
   <div style="width:100%;" class="NavContent">
 
   <div style="width:100%;" class="NavContent">
 
     <p style="text-align: left;">
 
     <p style="text-align: left;">
problem solution </p>
+
From the normalizing condition  $V^2=\xi^\mu \xi_\mu$. Here we will use a different meaning of acceleration, natural in the context of GTR. On a geodesic the covariant acceleration of a particle $w^\mu = u^\mu \nabla_{\mu} u^\nu$ is zero; if a particle moves not on a geodesic, this means that some (non-gravitational) force (per unit mass) is acting on it, equal to $w^{\mu}=-a^\mu$.
 +
 
 +
First of all, let us note that, as $\nabla g=0$,
 +
\[\xi^{\mu}\nabla_{\mu}V
 +
=\xi^{\mu}\xi^{\nu}
 +
(\nabla_{\mu}\xi_{\nu}+\nabla_{\nu}\xi_{\mu})=0,\]
 +
and also
 +
\[2V\nabla_{\mu}V=\nabla_{\mu}V^2
 +
=\nabla_{\mu}\xi^{\nu}\xi_{\nu}
 +
=2\xi^{\nu}\nabla_{\mu}\xi_{\nu}.\]
 +
Then using stationarity $u^\mu=\xi_\mu /V$ and the Killing equation, we get
 +
\[ -a^\mu = w^\mu=\frac{1}{V}\xi^{\nu}\nabla_{\nu}
 +
\big(\frac{1}{V}\xi^\mu\big)
 +
=\frac{1}{V^2}\xi_{\nu}\nabla^{\nu}\xi^{\mu}
 +
=-\frac{1}{V^2}\xi_{\nu}\nabla^{\mu}\xi^{\nu}
 +
=-\frac{1}{V}\nabla^{\mu}V=-\nabla^{\mu}\ln V,\]
 +
therefore
 +
\[a^{\mu}=\nabla^{\mu}\Phi,\quad\text{where}
 +
\quad \Phi=\ln V=\tfrac{1}{2}\ln \xi^{\mu}\xi_{\nu}.\]
 +
 
 +
In the weak field limit $g_{00}\approx 1+\tfrac{2\phi}{c^2}$, so $\Phi\approx \phi/c^2$. $\phi$ is the Newtonian gravitational potential. </p>
 
   </div>
 
   </div>
 
</div>
 
</div>
Line 860: Line 943:
 
<div id="BlackHole43"></div>
 
<div id="BlackHole43"></div>
 
=== Problem 30. ===
 
=== Problem 30. ===
problem formulation
+
Surface gravity $\kappa$ of the Schwarzschild horizon can be defined as acceleration of a stationary particle at the horizon, measured in the proper time of a stationary observer at infinity. Find $\kappa$.
 
<div class="NavFrame collapsed">
 
<div class="NavFrame collapsed">
 
   <div class="NavHead">solution</div>
 
   <div class="NavHead">solution</div>
 
   <div style="width:100%;" class="NavContent">
 
   <div style="width:100%;" class="NavContent">
 
     <p style="text-align: left;">
 
     <p style="text-align: left;">
problem solution </p>
+
The scalar acceleration near the horizon, measured in proper time, is
 +
\begin{align*}
 +
a=&\sqrt{|a^{\mu}a_{\mu}|}
 +
=\sqrt{|g^{\mu\nu}a_{\mu}a_{\nu}|}
 +
=\sqrt{|g^{11}|(a_1)^{1}}=\sqrt{|g^{11}|(\Phi')^2}=\\
 +
&=\sqrt{g_{00}}\;\Phi'(r)
 +
=\sqrt{g_{00}}\;\frac{g_{00}'}{g_{00}}
 +
=\frac{g_{00}'}{2\sqrt{g_{00}}}.
 +
\end{align*}
 +
It tends to infinity at the horizon. The time of remote observer $t$ is related to the proper time as $dt=d\tau\sqrt{g_{00}}$, therefor acceleration measured in time $t$ is
 +
\[a_{\infty}=\sqrt{g_{00}}\;a=\frac{1}{2}g_{00}',\]
 +
and it is finite. At $r=r_{g}$ it gives us the surface gravity
 +
\[\kappa=a_{\infty}\Big|_{r=r_g}=\frac{1}{2r_{g}}
 +
=\frac{c^4}{4MG}.\]
 +
In the last equality we restored the dimensional factor $c^2$. </p>
 
   </div>
 
   </div>
 
</div>
 
</div>
Line 876: Line 973:
 
<div id="BlackHole44"></div>
 
<div id="BlackHole44"></div>
 
=== Problem 31. ===
 
=== Problem 31. ===
problem formulation
+
Suppose all the matter is distributed around the center of symmetry, and its energy-momentum tensor is spherically symmetric, so that the form of $g_{\mu\nu}$ written above is correct. Show that the solution in the exterior region is reduced to the Schwarzschild metric and find the relation between $C$ and the system's mass $M$.
 
<div class="NavFrame collapsed">
 
<div class="NavFrame collapsed">
 
   <div class="NavHead">solution</div>
 
   <div class="NavHead">solution</div>
 
   <div style="width:100%;" class="NavContent">
 
   <div style="width:100%;" class="NavContent">
 
     <p style="text-align: left;">
 
     <p style="text-align: left;">
problem solution </p>
+
We eliminate the factor $f$ in $g_{00}$ by coordinate transformation $\sqrt{f(t)}dt=dt'$; other components of the metric do not change. In the weak field limit  $g_{00}\approx(1+\tfrac{2\varphi}{c^2})$, where  $\varphi=-GM/r$ is the Newtonian gravitational potential. Comparing with the asymptote of $g_{00}$ we get
 +
\[C=r_{g}=\frac{2GM}{c^2}.\]</p>
 
   </div>
 
   </div>
 
</div>
 
</div>
Line 887: Line 985:
 
<div id="BlackHole45"></div>
 
<div id="BlackHole45"></div>
 
=== Problem 32. ===
 
=== Problem 32. ===
problem formulation
+
Let there be a spherically symmetric void $r<r_{0}$ in the spherically symmetric matter distribution. Show that spacetime in the void is flat.
 
<div class="NavFrame collapsed">
 
<div class="NavFrame collapsed">
 
   <div class="NavHead">solution</div>
 
   <div class="NavHead">solution</div>
 
   <div style="width:100%;" class="NavContent">
 
   <div style="width:100%;" class="NavContent">
 
     <p style="text-align: left;">
 
     <p style="text-align: left;">
problem solution </p>
+
The integration constant is determined by the demand of boundedness of $g_{\mu\nu}$ in the region: $C=0$. Then on coordinate transformation $f(t)dt^2=d\tau^2$ we will obtain the flat Minkowskii metric
 +
\[ds^2=d\tau^2-dr^2-r^2 d\Omega^2.\] </p>
 
   </div>
 
   </div>
 
</div>
 
</div>
Line 898: Line 997:
 
<div id="BlackHole46"></div>
 
<div id="BlackHole46"></div>
 
=== Problem 33. ===
 
=== Problem 33. ===
problem formulation
+
Let the matter distribution be spherically symmetric and filling regions  $r<r_{0}$ and $r_{1}<r<r_{2}$ ($r_{0}<r_{1}$). Can one affirm, that the solution in the layer of empty space $r_{0}<r<r_{1}$ is also the Schwarzschild metric?
 
<div class="NavFrame collapsed">
 
<div class="NavFrame collapsed">
 
   <div class="NavHead">solution</div>
 
   <div class="NavHead">solution</div>
 
   <div style="width:100%;" class="NavContent">
 
   <div style="width:100%;" class="NavContent">
 
     <p style="text-align: left;">
 
     <p style="text-align: left;">
problem solution </p>
+
With such matter distribution the solution in region $r>r_{2}$ is Schwarzschild, as the constant $C$ and function $f(t)$ are fixed in the usual way. In region $r_{0}<r<r_{1}$, however, we do not have the freedom to choose a new time coordinate, as it will be determined by the smooth sewing-up of the metric at the boundaries (in region $r_{1}<r<r_{2}$ the solution is quite different, of course). Therefore $f(t)\neq 1$ in the inner region. </p>
 
   </div>
 
   </div>
 
</div>
 
</div>

Revision as of 20:25, 17 June 2012


The spherically symmetric solution of Einstein's equations in vacuum for the spacetime metric has the form \cite{Schw} \begin{align}\label{Schw} ds^{2}=h(r)\,dt^2-h^{-1}(r)\,dr^2-r^2 d\Omega^{2}, &\qquad\mbox{where}\quad h(r)=1-\frac{r_g}{r};\quad r_{g}=\frac{2GM}{c^{2}};\\ d\Omega^{2}=d\theta^{2}+\sin^{2}\theta\, d\varphi^{2}&\;\text{-- metric of unit sphere.}\nonumber \end{align} The Birkhoff's theorem (1923) \cite{Birkhoff,Jebsen} states, that this solution is unique up to coordinate transformations. The quantity $r_g$ is called the Schwarzschild radius, or gravitational radius, $M$ is the mass of the central body or black hole.

Simple problems

Problem 1.

Find the interval of local time (proper time of stationary observer) at a point $(r,\theta,\varphi)$ in terms of coordinate time $t$, and show that $t$ is the proper time of an observer at infinity. What happens when $r\to r_{g}$?

Problem 2.

What is the physical distance between two points with coordinates $(r_{1},\theta,\varphi)$ and $(r_{2},\theta,\varphi)$? Between $(r,\theta,\varphi_{1})$ and $(r,\theta,\varphi_{2})$? How do these distances behave in the limit $r_{1},r\to r_{g}$?

Problem 3.

What would be the answers to the previous two questions for $r<r_g$ and why*? Why the Schwarzschild metric cannot be imagined as a system of "welded" rigid rods in $r<r_g$, as it can be in the external region?


Problem 4.

Calculate the acceleration of a test particle with zero velocity.

Problem 5.

Show that Schwarzschild metric is a solution of Einstein's equation in vacuum.

Symmetries and integrals of motion

For background on Killing vectors see problems K1, K2, K3 of chapter 2.

Problem 6.

What integral of motion arises due to existance of a timelike Killing vector? Express it through the physical velocity of the particle.

Problem 7.

Derive the Killing vectors for a sphere in Cartesian coordinate system; in spherical coordinates.

Problem 8.

Verify that in coordinates $(t,r,\theta,\varphi)$ vectors \[ \begin{array}{l} \Omega^{\mu}=(1,0,0,0),\\ R^{\mu}=(0,0,0,1),\\ S^{\mu}=(0,0,\cos\varphi,-\cot\theta\sin\varphi),\\ T^{\mu}=(0,0,-\sin\varphi,-\cot\theta\cos\varphi) \end{array}\] are the Killing vectors of the Schwarzschild metric.

Problem 9.

Show that existence of Killing vectors $S^\mu$ and $T^\mu$ leads to motion of particles in a plane.

Problem 10.

Show that the particles' motion in the plane is stable.

Problem 11.

Write down explicitly the conserved quantities $p_{\mu}\Omega^{\mu}$ and $p_{\mu}R^{\mu}$ for movement in the plane $\theta=\pi/2$.

Problem 12.

What is the work needed to pull a particle from the horizon to infinity? Will a black hole's mass change if we drop a particle with zero initial velocity from immediate proximity of the horizon?

Radial motion

Consider a particle's radial motion: $\dot{\varphi}=\dot{\theta}=0$. In this problem one is especially interested in asymptotes of all functions as $r\to r_{g}$.

Let us set $c=1$ here and henceforth measure time in the units of length, so that $x^{0}=t$, $\beta=v$, etc., and introduce the notation \[h(r)\equiv g_{00}(r)=-\frac{1}{g_{11}(r)}= 1-\frac{r_{g}}{r} \underset{r\to r_g+0}{\longrightarrow}+0.\]

Problem 13.

Derive the equation for null geodesics (worldlines of massless particles).

Problem 14.

Use energy conservation to derive $v(r)$, $\dot{r}(r)=dr/dt$, $r(t)$ for a massive particle. Initial conditions: $g_{00}|_{\dot{r}=0}=h_{0}$ (the limiting case $h_{0}\to 1$ is especially interesting and simple).

Problem 15.

Show that the equation of radial motion in terms of proper time of the particle is the same as in the non-relativistic Newtonian theory. Calculate the proper time of the fall from $r=r_0$ to the center. Derive the first correction in $r_{g}/r$ to the Newtonian result. Initial velocity is zero.

Problem 16.

Derive the equations of radial motion in the ultra-relativistic limit.

Problem 17.

A particle (observer) falling into a black hole is emitting photons, which are detected on the same radial line far away from the horizon (i.e. the photons travel from emitter to detector radially). Find $r$, $v$ and $\dot{r}$ as functions of the signal's detection time in the limit $r\to r_g$.

Blackness of black holes

A source radiates photons of frequency $\omega_i$, its radial coordinate at the time of emission is $r=r_{em}$. Find the frequency of photons registered by a detector situated at $r=r_{det}$ on the same radial line in different situations described below. By stationary observers here, we mean stationary in the static Schwarzschild metric; "radius" is the radial coordinate $r$.

Problem 18.

The source and detector are stationary.

Problem 19.

The source is falling freely without initial velocity from radius $r_0$; it flies by the stationary detector at the moment of emission.

Problem 20.

The source is freely falling the same way, while the detector is stationary at $r_{det}>r_{em}$.

Problem 21.

The source is falling freely and emitting continuously photons with constant frequency, the detector is stationary far away from the horizon $r_{det}\gg r_{g}$. How does the detected light's intensity depend on $r_{em}$ at the moment of emission? On the proper time of detector?

Orbital motion, effective potential

Due to high symmetry of the Schwarzschild metric, a particle's worldline is completely determined by the normalizing condition $u^{\mu}u_{\mu}=\epsilon$, where $\epsilon=1$ for a massive particle and $\epsilon=0$ for a massless one, plus two conservation laws---of energy and angular momentum.

Problem 22.

Show that the ratio of specific energy to specific angular momentum of a particle equals to $r_{g}/b$, where $b$ is the impact parameter at infinity (for unbounded motion).

Problem 23.

Derive the geodesics' equations; bring the equation for $r(\lambda)$ to the form \[\frac{1}{2}\Big(\frac{dr}{d\lambda}\Big)^{2} +V_{\epsilon}(r)=\varepsilon,\] where $V_{\epsilon}(r)$ is a function conventionally termed as effective potential.

Problem 24.

Plot and investigate the function $V(r)$. Find the radii of circular orbits and analyze their stability; find the regions of parameters $(E,L)$ corresponding to bound and unbound motion, fall into the black hole. Consider the cases of a) massless, b) massive particles.

Problem 25.

Derive the gravitational capture cross-section for a massless particle; the first correction to it for a massive particle ultra-relativistic at infinity. Find the cross-section for a non-relativistic particle to the first order in $v^2/c^2$.

Problem 26.

Find the minimal radius of stable circular orbit and its parameters. What is the maximum gravitational binding energy of a particle in the Schwarzschild spacetime?

Miscellaneous problems

Problem 27.

Gravitational lensing is the effect of deflection of a light beam's (photon's) trajectory in the gravitational field. Derive the deflection of a photon's trajectory in Schwarzschild metric in the limit $L/r_{g}\gg 1$. Show that it is twice the value for a massive particle with velocity close to $c$ in the Newtonian theory.

Problem 28.

Show that the $4$-acceleration of a stationary particle in the Schwarzschild metric can be presented in the form \[a_{\mu}=-\partial_{\mu}\Phi,\quad \text{where}\quad \Phi=\ln \sqrt{g_{00}} =\tfrac{1}{2}\ln g_{00}\] is some generalization of the Newtonian gravitational potential.

Problem 29.

Let us reformulate the problem in a coordinate-independent manner. Suppose we have an arbitrary stationary metric with timelike Killing vector $\xi^\mu$, and we denote the $4$-velocity of a stationary observer by $u^{\mu}=\xi^{\mu}/V$. What is the $4$-force per unit mass that we need to apply to a test particle in order to make it stay stationary? Show in coordinate-independent way that the answer coincides with $\partial_{\mu}\Phi$ (up to the sign), and rewrite $\Phi$ in coordinate-independent form.

Problem 30.

Surface gravity $\kappa$ of the Schwarzschild horizon can be defined as acceleration of a stationary particle at the horizon, measured in the proper time of a stationary observer at infinity. Find $\kappa$.

Solving Einstein's equations for a spherically symmetric metric of general form in vacuum (energy-momentum tensor equals to zero), one can reduce the metric to \[ds^2=f(t)\Big(1-\frac{C}{r}\Big)dt^2 -\Big(1-\frac{C}{r}\Big)^{-1}dr^2-r^2 d\Omega^2,\] where $C$ is some integration constant, and $f(t)$ an arbitrary function of time $t$.

Problem 31.

Suppose all the matter is distributed around the center of symmetry, and its energy-momentum tensor is spherically symmetric, so that the form of $g_{\mu\nu}$ written above is correct. Show that the solution in the exterior region is reduced to the Schwarzschild metric and find the relation between $C$ and the system's mass $M$.

Problem 32.

Let there be a spherically symmetric void $r<r_{0}$ in the spherically symmetric matter distribution. Show that spacetime in the void is flat.

Problem 33.

Let the matter distribution be spherically symmetric and filling regions $r<r_{0}$ and $r_{1}<r<r_{2}$ ($r_{0}<r_{1}$). Can one affirm, that the solution in the layer of empty space $r_{0}<r<r_{1}$ is also the Schwarzschild metric?

Different coordinates, maximal extension

We saw that a particle's proper time of reaching the singularity is finite. However, the Schwarzschild metric has a (removable) coordinate singularity at $r=r_{g}$. In order to eliminate it and analyze the casual structure of the full solution, it is convenient to use other coordinate frames. Everywhere below we transform the coordinates $r$ and $t$, while leaving the angular part unchanged.

Problem 34.

problem formulation

Problem 35.

problem formulation

Problem 36.

problem formulation

Problem 37.

problem formulation

Problem 38.

problem formulation

Problem 39.

problem formulation